Math Problem Statement

Which one of the following is the subset of set = {x; where x is an prime number less than 10} ?

Choose at least one correct answer

Question 5Answer

a. {x; where x is an odd number less than 5}

b. {x; where x is a positive integers with less than 3 factors}

c. {x; where x is an odd number less than 10}

d. {x; Where x∈W & 1<x<4}

Solution

Ask a new question for Free

By Image

Drop file here or Click Here to upload

Math Problem Analysis

Mathematical Concepts

Set Theory
Prime Numbers
Subsets

Formulas

-

Theorems

-

Suitable Grade Level

High School